PT65, S1 LR, #16 Forum

Prepare for the LSAT or discuss it with others in this forum.
Post Reply
jawu9

New
Posts: 1
Joined: Sat Sep 29, 2012 9:58 pm

PT65, S1 LR, #16

Post by jawu9 » Sat Sep 29, 2012 10:04 pm

The correct answer is B, but I think D is completely viable!

I interpreted "But if all the members abstain, the matter will be decided by the city's voters" as an if --> then statement. We know that all abstentions will cause the voters to decide. But the stimulus never explicitly states that one council vote will prevent the proposal from being decided by voters. What if the matter will be decided by voters even if some members vote? Answer choice D would bridge that assumption.

The stimulus seems to imply that the voters will not decide if at least one vote is cast, but there is not enough information to infer that! We know all abstentions is sufficient to cause the matter to be decided by voters, but there could be other sufficient causes.

Can someone clear this up for me?

User avatar
CalAlumni

Bronze
Posts: 204
Joined: Fri May 11, 2012 11:58 am

Re: PT65, S1 LR, #16

Post by CalAlumni » Sat Sep 29, 2012 10:56 pm

jawu9 wrote:The correct answer is B, but I think D is completely viable!

I interpreted "But if all the members abstain, the matter will be decided by the city's voters" as an if --> then statement. We know that all abstentions will cause the voters to decide. But the stimulus never explicitly states that one council vote will prevent the proposal from being decided by voters. What if the matter will be decided by voters even if some members vote? Answer choice D would bridge that assumption.

The stimulus seems to imply that the voters will not decide if at least one vote is cast, but there is not enough information to infer that! We know all abstentions is sufficient to cause the matter to be decided by voters, but there could be other sufficient causes.

Can someone clear this up for me?
This is how I approach the question.

Before reading the passage, I glance at the question to identify the type of question being asked so I know what I'm looking for. Upon doing that, I notice that it is a JUSTIFY THE CONCLUSION question (LRB terminology).

Upon reading the passage it becomes apparent that the conclusion is the last line:

"So at least one member of the city council should vote against the proposal."

The goal is to JUSTIFY the conclusion. In other words, we need to add something to the passage that will make the conclusion 100% valid.

If either the city council ought to either vote against the proposal or abstain [okay so their giving us two choices for the city council]....but if all the members abstain, the matter will be decided by the city's voters....[ummm okay that is self explanatory]...[conclusion] -->So at least one member of the city council should vote against the proposal [ummm, how does this follow...what condition if added would justify this conclusion...B--"The proposal should not be decided by the city's voters." See, we don't want to give the choice to the city's voters...as the conclusion indicates...so adding answer choice B would be sufficient for the conclusion to follow.

(D) is a conditional statement that doesn't make the conclusion sufficient....it's a true statement...but this is not a MBT question.

Post Reply

Return to “LSAT Prep and Discussion Forum”